www.vorhilfe.de
- Förderverein -
Der Förderverein.

Gemeinnütziger Verein zur Finanzierung des Projekts Vorhilfe.de.
Hallo Gast!einloggen | registrieren ]
Startseite · Mitglieder · Impressum
Forenbaum
^ Forenbaum
Status VH e.V.
  Status Vereinsforum

Gezeigt werden alle Foren bis zur Tiefe 2

Navigation
 Startseite...
 Suchen
 Impressum
Das Projekt
Server und Internetanbindung werden durch Spenden finanziert.
Organisiert wird das Projekt von unserem Koordinatorenteam.
Hunderte Mitglieder helfen ehrenamtlich in unseren moderierten Foren.
Anbieter der Seite ist der gemeinnützige Verein "Vorhilfe.de e.V.".
Partnerseiten
Weitere Fächer:

Open Source FunktionenplotterFunkyPlot: Kostenloser und quelloffener Funktionenplotter für Linux und andere Betriebssysteme
Forum "Physik" - Drehmoment
Drehmoment < Physik < Naturwiss. < Vorhilfe
Ansicht: [ geschachtelt ] | ^ Forum "Physik"  | ^^ Alle Foren  | ^ Forenbaum  | Materialien

Drehmoment: Frage (beantwortet)
Status: (Frage) beantwortet Status 
Datum: 11:09 Fr 04.04.2008
Autor: tim_tempel

Aufgabe
Ein rotierender Korper hat folgende Winkel-Zeit-Funktion: ß =[mm] 35 + 12 t s^{-1} - 4 t^{2} s^{-2}[/mm] tsgegeben: J = 3 kg [mm] m^{2} [/mm]  
a) Wie groß war seine Winkelgeschwindigkeit bei t = 0?
b) Welches Drehmoment wirkt auf ihn?

Hallo,

die Ergebnisse habe ich:

[Dateianhang nicht öffentlich]

Komme mit b nicht wirklich zurecht.
Die Leistung ist Drehmoment * Winkelgeschwindigkeit und daraus folgt:
M ist ja wieder das Drehmoment = die Ableitung der Leistung?

Wäre nett, wenn mir jemand helfen kann.

Gruß, Tim

Dateianhänge:
Anhang Nr. 1 (Typ: png) [nicht öffentlich]
        
Bezug
Drehmoment: Drehimpuls
Status: (Antwort) fertig Status 
Datum: 11:31 Fr 04.04.2008
Autor: rainerS

Hallo!

> Ein rotierender Korper hat folgende Winkel-Zeit-Funktion: ß
> =[mm] 35 + 12 t s^{-1} - 4 t^{2} s^{-2}[/mm] tsgegeben: J = 3 kg
> [mm]m^{2}[/mm]  
> a) Wie groß war seine Winkelgeschwindigkeit bei t = 0?
> b) Welches Drehmoment wirkt auf ihn?
>
> Hallo,
>  
> die Ergebnisse habe ich:
>  
> [Dateianhang nicht öffentlich]
>  
> Komme mit b nicht wirklich zurecht.
> Die Leistung ist Drehmoment * Winkelgeschwindigkeit und
> daraus folgt:
>  M ist ja wieder das Drehmoment = die Ableitung der
> Leistung?

L ist der Drehimpuls, nicht die Leistung! Und der ist gleich dem Produkt aus Trägheitsmoment J und WInkelgeschwindigkeit.

Viele Grüße
   Rainer

Bezug
Ansicht: [ geschachtelt ] | ^ Forum "Physik"  | ^^ Alle Foren  | ^ Forenbaum  | Materialien


^ Seitenanfang ^
ev.vorhilfe.de
[ Startseite | Mitglieder | Impressum ]